0% found this document useful (0 votes)
823 views5 pages

IOQM 2021 Question Paper With Solutions

The document contains 12 problems with solutions related to mathematics. Some key details: - Problem 1 proves that four points related to two triangles are concyclic. - Problem 2 finds all natural numbers n for which a permutation of numbers satisfies a given condition. - Problem 3 involves counting arrangements of numbers that satisfy ordering conditions, and relates the count to powers of 2.

Uploaded by

Shristi Yadav
Copyright
© © All Rights Reserved
We take content rights seriously. If you suspect this is your content, claim it here.
Available Formats
Download as PDF, TXT or read online on Scribd
0% found this document useful (0 votes)
823 views5 pages

IOQM 2021 Question Paper With Solutions

The document contains 12 problems with solutions related to mathematics. Some key details: - Problem 1 proves that four points related to two triangles are concyclic. - Problem 2 finds all natural numbers n for which a permutation of numbers satisfies a given condition. - Problem 3 involves counting arrangements of numbers that satisfy ordering conditions, and relates the count to powers of 2.

Uploaded by

Shristi Yadav
Copyright
© © All Rights Reserved
We take content rights seriously. If you suspect this is your content, claim it here.
Available Formats
Download as PDF, TXT or read online on Scribd
You are on page 1/ 5

IOQM 2021-22 Part A

1. Three parallel lines L1 , L2 , L3 are drawn in the plane such that the perpendicular distance
between L1 and L2 is 3 and the perpendicular distance between L2 and L3 is also 3. A
square ABCD is constructed such that A lies on L1 , B lies on L3 and C lies on L2 . Find
the area of the square.

2. Ria writes down the numbers 1, 2, . . . , 101 in red and blue pens. The largest blue number
is equal to the number of numbers written in blue and the smallest red number is equal to
half the number of numbers written in red. How many numbers did Ria write with red pen?

3. Consider the set T of all triangles whose sides are distinct prime numbers which are also
in arithmetic progression. Let 4 ∈ T be the triangle with the least perimeter. If α ◦ is the
α
largest angle of 4 and if L is its perimeter, determine the value of .
L
4. Consider the set of all 6-digit numbers consisting of only 3 digits, a, b, c , where a, b, c are
distinct. Suppose the sum of all these numbers is 593999406. What is the largest remainder
when the three digit number abc is divided by 100 ?

5. In parallelogram ABCD the longer side is twice the shorter side. Let XYZW be the
quadrilateral formed by the internal bisectors of the angles of ABCD . If the area of XYZW
is 10, find the area of ABCD .

6. Let x, y, z be positive real numbers such that x 2 + y2 = 49 , y2 + yz + z 2 = 36 and


√ √ √
x2 + 3xz + z 2 = 25 . If the value of 2xy + 3yz + zx can be written as p q where p, q are
integers and q is not divisible by square of any prime number, find p + q .

7. Find the number of maps f : {1, 2, 3} −→ {1, 2, 3, 4, 5} such that f (i ) ≤ f (j) whenever i < j .

8. For any real number t , let bt c denote the largest integer ≤ t . Suppose that N is the
greatest integer such that  r q 
 j √ k
 N  = 4

Find the sum of digits of N .

9. Let P0 = (3, 1) and define Pn +1 = (xn , yn ) for n ≥ 0 by


3xn − yn xn + yn
xn +1 = − , yn +1 = −
2 2
Find the area of the quadrilateral formed by the points P96 , P97 , P98 , P99 .

10. Suppose that P is the polynomial of least degree with integer coefficients such that
√ √  √ √ 
P 7+ 5 =2 7− 5 . Find P (2) .

11. In how many ways can four married couples sit in a merry-go-round with identical seats
such that men and women occupy alternate seats and no husband seats next to his wife?

12. A 12 × 12 board is divided into 144 unit squares by drawing lines parallel to the sides. Two
rooks placed on two unit squares are said to be non attacking if they are not in the same
column or same row. Find the least number N such that if N rooks are placed on the unit
squares, one rook per square, we can always find 7 rooks such that no two are attacking
each other.
Question No. Answer
1 45
2 68
3 08
4 98
5 40
6 30
7 35
8 24
9 08
10 40
11 12
12 73
IOQM 2022 Part B
Official Solutions

Problem 1. Let D be an interior point on the side BC of an acute-angled triangle ABC.


Let the circumcircle of triangle ADB intersect AC again at E(̸= A) and the circumcircle of
triangle ADC intersect AB again at F (̸= A). Let AD, BE and CF intersect the circumcircle
of triangle ABC again at D1 (̸= A), E1 (̸= B) and F1 (̸= C), respectively. Let I and I1 be the
incentres of triangles DEF and D1 E1 F1 , respectively. Prove that E, F , I, I1 are concyclic.

E1
A
F1
F E

I I1

B D C

D1

Solution. Note that

∠CF1 D1 = ∠CAD1 = ∠EAD = ∠EBD = ∠E1 BC = ∠E1 F1 C,

so F1 C is the bisector of ∠D1 E1 F1 . Similarly, E1 B is the bisector of ∠D1 E1 F1 , implying


I1 = BE1 ∩ CF1 . Now,

∠EDF = ∠EDA + ∠F DA = ∠EBA + ∠F CA


= ∠E1 BA + ∠F1 CA = ∠E1 D1 A + ∠F1 D1 A = ∠E1 D1 F1 .

Therefore
1 1
∠EIF = 90◦ + ∠EDF = 90◦ + ∠E1 D1 F1 = ∠E1 I1 F1 = ∠EI1 F,
2 2
which proves the required concyclicity.

Problem 2. Find all natural numbers n for which there exists a permutation σ of 1, 2, . . . , n
such that
Xn
σ(i)(−2)i−1 = 0.
i=1

Note: A permutation of 1, 2, . . . , n is a bijective function from {1, 2, . . . , n} to itself.

1
Solution. Suppose that n ≡ 1 (mod 3) and σ a permutation of 1, 2, . . . , n. Then
n n
X X n(n + 1)
σ(i)(−2)i−1 ≡ σ(i) = (mod 3),
i=1 i=1
2

and hence the left-hand side is non-zero.


We now show by induction that if n ≡ 0 or 2 (mod 3) then there exists a permutation
of 1, 2, . . . , n satisfying the given condition.
If n = 2 then the permutation given by σ(1) = 2, σ(2) = 1 satisfies the given condition.
Similarly, if n = 3 then the permutation σ(1) = 2, σ(2) = 3, σ(3) = 1 satisfies the given
condition.
Suppose that for n = m there exists a permutation σ satisfying the given condition. We
consider the permutation τ of 1, 2, . . . , m + 3 given by τ (1) = 2, τ (2) = 3, τ (m + 3) = 1 and
τ (i) = σ(i − 2) + 3 for i = 3, 4, . . . , m + 2. Then
m+3
X m+2
X
i−1 m+2
τ (i)(−2) = 2 − 6 + (−2) + 3 · (−2)i−1
i=1 i=3
= 2 − 6 + (−2)m+2 − 4 · ((−2)m − 1) = 0 .

Thus, by induction it follows that for every n ≡ 0 or 2 (mod 3) there exists a permuta-
tion satisfying the given condition.

Problem 3. For a positive integer N , let T (N ) denote the number of arrangements of the
integers 1, 2, . . . , N into a sequence a1 , a2 , . . . , aN such that ai > a2i for all i, 1 ≤ i < 2i ≤ N
and ai > a2i+1 , for all i, 1 ≤ i < 2i + 1 ≤ N . For example, T (3) is 2, since the possible
arrangements are 321 and 312.
(a) Find T (7).

(b) If K is the largest non-negative integer so that 2K divides T (2n − 1), show that K =
2n − n − 1.
(c) Find the largest non-negative integer K so that 2K divides T (2n + 1).

Solution. (a) Given an arrangement a1 , a2 , . . . , a7 , satisfying the given conditions, we can


build a binary tree with nodes as in the Figure below. At each node, the root node

is greater than the child nodes. Conversely, any such tree gives a valid arrangement.
Observing that the root of the tree must 
contain
 the maximum of the numbers, we can
6
choose 3 out of the other 6 numbers in ways and build the left tree and the right
3  
6
tree, each in 2 ways and hence the number of such trees is 2 · 2 · = 80.
3

(b) Observe that T (N ) is also the number of ways of arranging any N distinct numbers
into a sequence a1 , a2 , . . . , aN satisfying the given conditions. Also, the given conditions
imply that a1 = maximum of the numbers. Now, leaving out the maximum, the rest of
the 2n − 2 numbers can be split into two groups of 2n−1 − 1 numbers each and these can
be individually put into a sequences b1 , b2 . . . , b2n−1 −1 and c1 , c2 , . . . , c2n−1 −1 satisfying the

2
conditions in T (n − 1) ways each. Now, the required arrangement of the original given
sequence can be obtained as follows:

a1 , b1 , c1 , b2 , b3 , c2 , c3 , b4 , b5 , b6 , b7 , c4 , c5 , c6 , c7 , . . .

This gives

2n − 2
 
T (2n − 1) = T (2n−1 − 1)2
n−1
(1)
2 −1
 n 
2 −2
We find the highest power of 2 that divides :
2n−1 − 1

We have
2n 2n !
 
n−2
2 = 2n−2 ·
2n−1 2n−1 !2n−1 !
2n (2n − 1)(2n − 2)!
= 2n−2 · n−1 n−1
2 (2 − 1)!2n−1 (2n−1 − 1)!
 n 
n 2 −2
= (2 − 1) n−1
2 −1
 n 
2
Now, the highest power of 2 that divides is
2n−1

2n−1 + 2n−2 + · · · + 1 − 2 2n−2 + 2n−3 + · · · + 1 = 1


 

 n 
2 −2
Hence the highest power of 2 that divides is n − 1.
2n−1 − 1

From the recurrence (1), if tn is the highest power of 2 dividing T (2n − 1), then tn =
2tn−1 + n − 1. From the initial conditions, t1 = 0, t2 = 1, t3 = 4, we obtain, by an easy
induction, that tn = 2n − n − 1.

(c) Suppose that N = 2n + 1. It is easy to see that

2n
 
T (2n + 1) = T (2n−1 − 1)T (2n−1 + 1) n−1
2 +1

2n
 
The highest power of 2 dividing is n:
2n−1 + 1

2n
  n 

2
(2n−1 + 1) = · 2n−1
2n−1 + 1 2n−1
 n 
2
Since the highest power of 2 dividing n−1
is 1, it follows that the highest power of 2
2
2n
 
dividing n−1
is n. Thus, if sn denotes the highest power of 2 dividing T (2n + 1),
2 +1
then

sn = sn−1 + 2n−1 − (n − 1) − 1 + n = sn−1 + 2n−1

Hence sn − s1 = 2n − 2 and since s1 = 1 (since T (3) = 2), it follows that the highest power
of 2 dividing T (2n + 1) is 2n − 1.

You might also like